June 2007 - Sec 2 - LR - Q9

Video Transcript:

0:06
question 9: Although video game sales have increased steadily over the past
0:11
three years we can expect a reversal of this trend in the very near future.
0:16
Historically over three quarters of video games sold have been purchased by
0:20
people from 13 to 16 years of age, and the number of people in this age group
0:25
is expected to decline steadily over the next ten years. Alright, so argument or facts? We have an
0:31
argument. What is this passage trying to convey? It's that we can expect a reversal
0:37
of this trend. What was this trend? That video game sales increased steadily over
0:42
the past three years. We can expect that to reverse in the very near future. And
0:47
how does the author prove that? Well, historically three quarters of video games sold
0:53
have been purchased by people from 13 to 16 years of age and the number of people
0:57
in this age group is expected to decline steadily over the next ten
1:01
years. So now that we have a clear understanding of this argument let's proceed to the
1:06
question stem. Which one of the following if true would most seriously weaken
1:10
the argument. Most seriously weaken... This is a weaken question. So we're trying to
1:16
weaken his argument that we can expect a reversal of this trend in the very near
1:21
future.
1:22
Again this trend being the increase in video game sales over the past few years.
1:26
And why? Because historically three quarters of video games have been bought by people from 13 to
1:33
16 years of age and the number of people in this age group is expected to decline
1:37
steadily over the next ten years. So let's take a look here at (A). Most people 17
1:43
years old or older have never purchased a video game. And you notice that if anything
1:49
(A) strengthens this argument by telling us people above this age do not buy
1:55
video games. So (A) is out. We are trying to weaken. (B) Video game rentals have declined over
2:04
the past 3 years.
2:06
Completely irrelevant. This argument is about sales and the fact that video game
2:10
rentals have declined over the past three years is entirely consistent with
2:14
the increase in video game sales
2:16
over that same period. So (B) does not help us and (B) would be eliminated. (C) New
2:24
technology will undoubtedly make entirely new entertainment options available over
2:29
the next 10 years. And again you notice if anything (C) strengthens this argument. Not only are
2:35
we gonna have less people between the ages of 13 and 16
2:40
who historically buy three quarters of video games but also new technologies
2:46
are going to come out with new entertainment options. So just another
2:49
reason to expect a reversal in this trend in the near future. So (C) would be
2:54
eliminated. (D) The number of different types of video games available is
2:58
unlikely to decrease in the near future. Again completely irrelevant. What does that have to do
3:03
with this argument? Which brings process of elimination to (E).
3:07
Most of the people who have purchased video games over the past three years are
3:11
over the age of 16. And you notice this clearly weakens this argument because over the
3:17
past few years when we saw this increase in video game sales most of the people
3:22
buying them were over sixteen. The author though looks to historical numbers and
3:28
says historically three quarters of video games have been purchased by people ages
3:31
13 to 16 but what we really care about is who was buying them over these past
3:37
three years where we saw an increase in sales.
3:42
And (E) tells us that there are people over the age of 16.
3:45
So the premise that the author gives us that this group of ages 13 to 16 is gonna decrease over the next
3:52
ten years doesn't really matter.
3:55
So it weakens the argument and (E) would be the correct answer.